Free PDF Download of CBSE Class 10 Maths Chapter 1 Real Numbers Multiple Choice Questions with Answers. MCQ Questions for Class 10 Maths with Answers was Prepared Based on Latest Exam Pattern. Students can solve NCERT Class 10 Maths Real Numbers MCQs with Answers to know their preparation level. https://www.cbselabs.com/mcq-questions-for-class-10-maths-real-numbers-with-solutions/

Class 10 Maths MCQs Chapter 1 Real Numbers

Real Numbers Class 10 MCQ

1. The decimal form of \(\frac{129}{2^{2} 5^{7} 7^{5}}\) is
(a) terminating
(b) non-termining
(c) non-terminating non-repeating
(d) none of the above

Answer

Answer: c


Class 10 Maths Chapter 1 MCQ

2. HCF of 8, 9, 25 is
(a) 8
(b) 9
(c) 25
(d) 1

Answer

Answer: d


Real Numbers MCQ Class 10

3. Which of the following is not irrational?
(a) (2 – √3)2
(b) (√2 + √3)2
(c) (√2 -√3)(√2 + √3)
(d)\(\frac{2 \sqrt{7}}{7}\)

Answer

Answer: c


Class 10 Real Numbers MCQ

4. The product of a rational and irrational number is
(a) rational
(b) irrational
(c) both of above
(d) none of above

Answer

Answer: b


MCQ Of Real Numbers Class 10

5. The sum of a rational and irrational number is
(a) rational
(b) irrational
(c) both of above
(d) none of above

Answer

Answer: b


Real Numbers MCQ

6. The product of two different irrational numbers is always
(a) rational
(b) irrational
(c) both of above
(d) none of above

Answer

Answer: b


MCQ On Real Numbers Class 10

7. The sum of two irrational numbers is always
(a) irrational
(b) rational
(c) rational or irrational
(d) one

Answer

Answer: a


Real Numbers Class 10 MCQ With Answers

8. If b = 3, then any integer can be expressed as a =
(a) 3q, 3q+ 1, 3q + 2
(b) 3q
(c) none of the above
(d) 3q+ 1

Answer

Answer: a


Real No Class 10 MCQ

9. The product of three consecutive positive integers is divisible by
(a) 4
(b) 6
(c) no common factor
(d) only 1

Answer

Answer: b


Class 10 Maths Real Numbers MCQ

10. The set A = {0,1, 2, 3, 4, …} represents the set of
(a) whole numbers
(b) integers
(c) natural numbers
(d) even numbers

<strongAnswer

Answer: a


11. Which number is divisible by 11?
(a) 1516
(b) 1452
(c) 1011
(d) 1121

Answer

Answer: b


12. LCM of the given number ‘x’ and ‘y’ where y is a multiple of ‘x’ is given by
(a) x
(b) y
(c) xy
(d) \(\frac{x}{y}\)

Answer

Answer: b


13. The largest number that will divide 398,436 and 542 leaving remainders 7,11 and 15 respectively is
(a) 17
(b) 11
(c) 34
(d) 45

Answer/ Explanation

Answer: a
Explaination:(a); [Hint. Algorithm 398 – 7 – 391; 436 – 11 = 425; 542 – 15 = 527; HCF of 391, 425, 527 = 17]


14. There are 312, 260 and 156 students in class X, XI and XII respectively. Buses are to be hired to take these students to a picnic. Find the maximum number of students who can sit in a bus if each bus takes equal number of students
(a) 52
(b) 56
(c) 48
(d) 63

Answer/ Explanation

Answer: a
Explaination:(a); [Hint. HCF of 312, 260, 156 = 52]


15. There is a circular path around a sports field. Priya takes 18 minutes to drive one round of the field. Harish takes 12 minutes. Suppose they both start at the same point and at the same time and go in the same direction. After how many minutes will they meet ?
(a) 36 minutes
(b) 18 minutes
(c) 6 minutes
(d) They will not meet

Answer/ Explanation

Answer: a
Explaination:(a); [Hint. LCM of 18 and 12 = 36]


16. Express 98 as a product of its primes
(a) 2² × 7
(b) 2² × 7²
(c) 2 × 7²
(d) 23 × 7

Answer

Answer: c


17. Three farmers have 490 kg, 588 kg and 882 kg of wheat respectively. Find the maximum capacity of a bag so that the wheat can be packed in exact number of bags.
(a) 98 kg
(b) 290 kg
(c) 200 kg
(d) 350 kg

Answer/ Explanation

Answer: a
Explaination:(a); [Hint. HCF of 490, 588, 882 = 98 kg]


18. For some integer p, every even integer is of the form
(a) 2p + 1
(b) 2p
(c) p + 1
(d) p

Answer

Answer: b


19. For some integer p, every odd integer is of the form
(a) 2p + 1
(b) 2p
(c) p + 1
(d) p

Answer

Answer: a


20. m² – 1 is divisible by 8, if m is
(a) an even integer
(b) an odd integer
(c) a natural number
(d) a whole number

Answer

Answer: b


21. If two positive integers A and B can be ex-pressed as A = xy3 and B = xiy2z; x, y being prime numbers, the LCM (A, B) is
(a) xy²
(b) x4y²z
(c) x4y3
(d) x4y3z

Answer

Answer: d


22. The product of a non-zero rational and an irrational number is
(a) always rational
(b) rational or irrational
(c) always irrational
(d) zero

Answer

Answer: c


23. If two positive integers A and B can be expressed as A = xy3 and B = x4y2z; x, y being prime numbers then HCF (A, B) is
(a) xy²
(b) x4y²z
(c) x4y3
(d) x4y3z

Answer

Answer: a


24. The largest number which divides 60 and 75, leaving remainders 8 and 10 respectively, is
(a) 260
(b) 75
(c) 65
(d) 13

Answer

Answer: d


25. The least number that is divisible by all the numbers from 1 to 5 (both inclusive) is
(a) 5
(b) 60
(c) 20
(d) 100

Answer

Answer: b
Explaination:(b); [Hint. LCM of 2, 3, 4, 5 = 60


26. The least number that is divisible by all the numbers from 1 to 8 (both inclusive) is
(a) 840
(b) 2520
(c) 8
(d) 420

Answer

Answer: a


27. The decimal expansion of the rational number \(\frac{14587}{250}\) will terminate after:
(a) one decimal place
(b) two decimal places
(c) three decimal places
(d) four decimal places

Answer

Answer: c


28. The decimal expansion of the rational number \(\frac{97}{2 \times 5^{4}}\) will terminate after:
(a) one decimal place
(b) two decimal places
(c) three decimal places
(d) four decimal places

Answer

Answer: d


29. The product of two consecutive natural numbers is always:
(a) prime number
(b) even number
(c) odd number
(d) even or odd

Answer

Answer: b


30. If the HCF of 408 and 1032 is expressible in the form 1032 x 2 + 408 × p, then the value of p is
(a) 5
(b) -5
(c) 4
(d) -4

Answer/ Explanation

Answer: b
Explaination:(b); [Hint. HCF of 408 and 1032 is 24, .-. 1032 x 2 + 408 x (-5)]


31. The number in the form of 4p + 3, where p is a whole number, will always be
(a) even
(b) odd
(c) even or odd
(d) multiple of 3

Answer

Answer: b


32. When a number is divided by 7, its remainder is always:
(a) greater than 7
(b) at least 7
(c) less than 7
(d) at most 7

Answer

Answer: c


33. (6 + 5 √3) – (4 – 3 √3) is
(a) a rational number
(b) an irrational number
(c) a natural number
(d) an integer

Answer

Answer: b


34. If HCF (16, y) = 8 and LCM (16, y) = 48, then the value of y is
(a) 24
(b) 16
(c) 8
(d) 48

Answer

Answer: a


35. According to the fundamental theorem of arith-metic, if T (a prime number) divides b2, b > 0, then
(a) T divides b
(b) b divides T
(c) T2 divides b2
(d) b2 divides T2

span style=”color: #ff00ff;”>Answer

Answer: a


36. The number ‘π’ is
(a) natural number
(b) rational number
(c) irrational number
(d) rational or irrational

Answer

Answer: c


37. If LCM (77, 99) = 693, then HCF (77, 99) is
(a) 11
(b) 7
(c) 9
(d) 22

Answer

Answer: a


38. Euclid’s division lemma states that for two positive integers a and b, there exist unique integer q and r such that a = bq + r, where r must satisfy
(a) a < r < b
(b) 0 < r ≤ b
(c) 1 < r < b
(d) 0 ≤ r < b

Answer

Answer: d


We hope the given MCQ Questions for Class 10 Maths Real Numbers with Answers will help you. If you have any query regarding CBSE Class 10 Maths Chapter 1 Real Numbers Multiple Choice Questions with Answers, drop a comment below and we will get back to you at the earliest.